Difference between revisions of "2002 AMC 10A Problems/Problem 4"

(see also and stuff)
 
Line 1: Line 1:
==Problem==
+
#redirect [[2002 AMC 12A Problems/Problem 6]]
For how many positive integers m is there at least 1 positive integer n such that <math>mn \le m + n</math>?
 
 
 
<math>\text{(A)}\ 4 \qquad \text{(B)}\ 6 \qquad \text{(C)}\ 9 \qquad \text{(D)}\ 12 \qquad \text{(E)}</math> Infinite.
 
 
 
==Solution==
 
We quickly see that for n=1, we have <math>m\le m+1</math>, so (m,1) satisfies the conditions for all m. Our answer is <math>\boxed{\text{(E) Infinite}}</math>.
 
 
 
==See Also==
 
{{AMC10 box|year=2002|ab=A|num-b=3|num-a=5}}
 
 
 
[[Category:Introductory Algebra Problems]]
 

Latest revision as of 07:56, 18 February 2009